matheraum.de
Raum für Mathematik
Offene Informations- und Nachhilfegemeinschaft

Für Schüler, Studenten, Lehrer, Mathematik-Interessierte.
Hallo Gast!einloggen | registrieren ]
Startseite · Forum · Wissen · Kurse · Mitglieder · Team · Impressum
Forenbaum
^ Forenbaum
Status Schulmathe
  Status Primarstufe
  Status Mathe Klassen 5-7
  Status Mathe Klassen 8-10
  Status Oberstufenmathe
    Status Schul-Analysis
    Status Lin. Algebra/Vektor
    Status Stochastik
    Status Abivorbereitung
  Status Mathe-Wettbewerbe
    Status Bundeswettb. Mathe
    Status Deutsche MO
    Status Internationale MO
    Status MO andere Länder
    Status Känguru
  Status Sonstiges

Gezeigt werden alle Foren bis zur Tiefe 2

Navigation
 Startseite...
 Neuerdings beta neu
 Forum...
 vorwissen...
 vorkurse...
 Werkzeuge...
 Nachhilfevermittlung beta...
 Online-Spiele beta
 Suchen
 Verein...
 Impressum
Das Projekt
Server und Internetanbindung werden durch Spenden finanziert.
Organisiert wird das Projekt von unserem Koordinatorenteam.
Hunderte Mitglieder helfen ehrenamtlich in unseren moderierten Foren.
Anbieter der Seite ist der gemeinnützige Verein "Vorhilfe.de e.V.".
Partnerseiten
Weitere Fächer:

Open Source FunktionenplotterFunkyPlot: Kostenloser und quelloffener Funktionenplotter für Linux und andere Betriebssysteme
StartseiteMatheForenUni-AnalysisVollständige Induktion
Foren für weitere Schulfächer findest Du auf www.vorhilfe.de z.B. Informatik • Physik • Technik • Biologie • Chemie
Forum "Uni-Analysis" - Vollständige Induktion
Vollständige Induktion < Analysis < Hochschule < Mathe < Vorhilfe
Ansicht: [ geschachtelt ] | ^ Forum "Uni-Analysis"  | ^^ Alle Foren  | ^ Forenbaum  | Materialien

Vollständige Induktion: Frage
Status: (Frage) beantwortet Status 
Datum: 12:57 Sa 30.04.2005
Autor: stevarino

Hallo

Ich hab ein Problem mit Vollständiger Induktion
[mm] 3^{n}>n^{3} [/mm]

Induktionsanfang für n=4 denn bis n=3 ist die Unlgleichung nicht erfüllt

[mm] 3^{4}>4^{3} [/mm]
81>64 stimmt

Induktionsannahme
[mm] 3^{n}>n^{3} [/mm]

Induktionsschluss für n=n+1

[mm] 3^{n+1}>(n+1)^{3} [/mm]
[mm] 3^n*3>n^3+3n^2+3n+1 [/mm] jetzt setz ich für [mm] 3^n n^3 [/mm] ein laut Annahme
[mm] 3^n*3>n^3*3>n^3+3n^2+3n+1 [/mm] jetzt häng ich wie gehts jetzt weiter ích müßte jetzt so umformen das daraus klar zu erkennen ist das [mm] n^3*3 [/mm] größer [mm] n^3+3n^2+3n+1 [/mm]  ist

vielen Dank schon mal

Stevo







        
Bezug
Vollständige Induktion: Antwort
Status: (Antwort) fertig Status 
Datum: 15:37 Sa 30.04.2005
Autor: FriedrichLaher

Hallo stevo

> Induktionsschluss für n=n+1
>  
> [mm]3^{n+1}>(n+1)^{3}[/mm]
>  [mm]3^n*3>n^3+3n^2+3n+1[/mm] jetzt setz ich für [mm]3^n n^3[/mm] ein laut
> Annahme

dividier nun einfach durch [mm] $3*3^n$ [/mm] - was ergibt sich da unter der Annahme
[mm] $3^n [/mm] > [mm] n^3$ [/mm] und n > 3 ?

Gru0 F.

Bezug
Ansicht: [ geschachtelt ] | ^ Forum "Uni-Analysis"  | ^^ Alle Foren  | ^ Forenbaum  | Materialien


^ Seitenanfang ^
www.schulmatheforum.de
[ Startseite | Forum | Wissen | Kurse | Mitglieder | Team | Impressum ]